OB Module 2: Care of the Pregnant Woman

Pataasin ang iyong marka sa homework at exams ngayon gamit ang Quizwiz!

A standard ultrasound examination is performed during the second or third trimester and includes an evaluation of which of the following? Note: Credit will be given only if all correct choices and no incorrect choices are selected. Select all that apply. 1. Confirm fetal heart activity. 2. Evaluate the cervix. 3. Determine fetal presentation. 4. Amniotic fluid volume. 5. Fetal number.

: 3, 4, 5 Explanation: 3. A standard ultrasound examination is performed during the second or third trimester and includes an evaluation to determine fetal presentation. 4. A standard ultrasound examination is performed during the second or third trimester and includes an evaluation of amniotic fluid volume. 5. A standard ultrasound examination is performed during the second or third trimester and includes an evaluation of fetal number.

) The prenatal clinic nurse is explaining test results to a client who has had an assessment for fetal well-being. Which statement indicates that the client understands the test result? 1. "The normal Doppler velocimetry wave result indicates my placenta is getting enough blood to the baby." 2. "The reactive non-stress test means that my baby is not growing because of a lack of oxygen." 3. "Because my contraction stress test was positive, we know that my baby will tolerate labor well." 4. "My biophysical profile score of 6 points to everything being normal and healthy for my baby."

Answer: 1 Explanation: 1. A decrease in fetal cardiac output or an increase in resistance of placental vessels will reduce umbilical artery blood flow. Doppler velocimetry is best used when intrauterine growth restriction is diagnosed; therefore, the baby is getting an adequate blood supply.

Which of the following tests provides information about the fetal number? 1. Amniocentesis 2. Standard second-trimester sonogram 3. Beta hCG 4. Maternal serum alpha-fetoprotein

Answer: 2 Explanation: 2. A standard (comprehensive) second trimester sonogram provides the information about the fetus, placenta, and uterine conditions including fetal number.

The client at 24 weeks' gestation is experiencing painless vaginal bleeding after intercourse. The physician has ordered a transvaginal ultrasound examination. Which statements by the client indicate an understanding of why this exam has been requested? Note: Credit will be given only if all correct choices and no incorrect choices are selected. Select all that apply. 1. "This ultrasound will show the baby's gender." 2. "This ultrasound might cause the miscarriage of my baby." 3. "This ultrasound carries a risk of creating a uterine infection." 4. "This ultrasound can determine the location of my placenta." 5. "This ultrasound might detect whether the placenta is detaching prematurely."

Answer: 4, 5 Explanation: 4. Painless bleeding in the second and third trimesters can be a symptom of placenta previa. Transvaginal ultrasound will determine the placental location. 5. Painless bleeding in the second and third trimesters can be a symptom of placenta previa. Transvaginal ultrasound will determine the placental location.

A client at 36 weeks' gestation is complaining of dyspnea when lying flat. What is the clinical reason for this complaint? 1. Maternal hypertension 2. Fundal height 3. Hydramnios 4. Congestive heart failure

Answer: 2 Explanation: 2. The dyspnea is resulting from the pressure of the enlarging uterus on the diaphragm.

When assisting with a transabdominal sampling, which of the following would the nurse do? 1. Obtain preliminary urinary samples. 2. Have the woman empty her bladder before the test begins. 3. Assist the woman into a supine position on the examining table. 4. Instruct the woman to eat a fat-free meal 2 hours before the scheduled test time.

Answer: 3 Explanation: 3. Clients are placed in a supine position on the table.

The nurse working in an outpatient obstetric clinic assesses four primigravida clients. Which client findings would the nurse tell the physician about? 1. 17 weeks' gestation and client denies feeling fetal movement 2. 24 weeks' gestation and fundal height is at the umbilicus 3. 4-6 weeks' gestation and softening of the cervix 4. 34 weeks' gestation and complains of hemorrhoidal pain

Answer: 2 Explanation: 2. The fundal height at 24 weeks should be 24 cm. The fundal height is usually at the umbilicus at 20-22 weeks.

In early-pregnancy class, the nurse emphasizes the importance of 8-10 glasses of fluid per day. How many of these should be water? 1. 1 to 2 2. 2 to 4 3. 4 to 6 4. 3 to 5

Answer: 3 Explanation: 3. A pregnant woman should consume at least 8 to 12 (8 oz) glasses of fluid each day, of which 4 to 6 glasses should be water.

The nurse notes the following findings in a client at 12-weeks' gestation. Which of the findings would enable the nurse to tell the client that she is diagnostically pregnant? 1. Fetal heart rate by Doppler 2. Positive pregnancy test 3. Positive Chadwicks sign 4. Montgomery gland enlargement

Answer: 1 Explanation: 1. A fetal heart rate by Doppler is a diagnostic (positive) change of pregnancy.

A 27-year-old married woman is 16 weeks pregnant and has an abnormally low maternal serum alpha-fetoprotein test. Which statement indicates that the couple understands the implications of this test result? 1. "We have decided to have an abortion if this baby has Down syndrome." 2. "If we hadn't had this test, we wouldn't have to worry about this baby." 3. "I'll eat plenty of dark green leafy vegetables until I have the ultrasound." 4. "The ultrasound should be normal because I'm under the age of 35."

Answer: 1 Explanation: 1. A low maternal serum alpha-fetoprotein test can indicate trisomy 18 or trisomy 21 (Down syndrome). Many couples abort a fetus that has a genetic abnormality that significantly affects quality of life or has multiple medical problems. Down syndrome is more likely to occur in the fetuses of women over the age of 35 at delivery, but is not limited to this age group.

The nurse is planning an educational session for pregnant vegans. What information should the nurse include? 1. Eating beans and rice provides complete protein needs. 2. Soy is not a good source of protein for vegans. 3. Rice contains a high level of vitamin B12. 4. Vegan diets are excessively high in iron.

Answer: 1 Explanation: 1. Adequate dietary protein can be obtained by consuming a varied diet with adequate caloric intake and plant-based proteins. Consuming an assortment of plant proteins throughout the day such as beans and rice, peanut butter on whole-grain bread, and whole-grain cereal with soy milk ensures that the expectant mother obtains all essential amino acids.

The nurse is preparing a client for amniocentesis. Which statement would indicate that the client clearly understands the risks of an amniocentesis? 1. "I might go into labor early." 2. "It could produce a congenital defect in my baby." 3. "Actually, there are no real risks to this procedure." 4. "The test could stunt my baby's growth."

Answer: 1 Explanation: 1. Amniocentesis has the potential for causing a spontaneous abortion.

The pregnant client and her partner are both 40 years old. The nurse is explaining the options of chorionic villus sampling (CVS) and amniocentesis for genetic testing. The nurse should correct the client if she makes which statement? 1. "Amniocentesis results are available sooner than CVS results are." 2. "CVS carries a higher risk of limb abnormalities." 3. "Amniocentesis cannot detect a neural tube defect." 4. "CVS is performed through my belly or my cervix."

Answer: 1 Explanation: 1. Amniocentesis results take longer to process than do CVS results.

A pregnant client who was of normal prepregnancy weight is now 30 weeks pregnant. She asks the nurse what appropriate weight gain for her should be. What is the nurse's best response? 1. "25-35 pounds" 2. "30-40 pounds" 3. "17-18 pounds" 4. "Less than 15 pounds"

Answer: 1 Explanation: 1. An appropriate weight gain for a woman of normal weight before pregnancy would be 25-35 pounds.

The nurse has completed a community presentation about the changes of pregnancy, and knows that the lesson was successful when a community member states that which of the following is one probable or objective change of pregnancy? 1. "Enlargement of the uterus" 2. "Hearing the baby's heart rate" 3. "Increased urinary frequency" 4. "Nausea and vomiting"

Answer: 1 Explanation: 1. An examiner can perceive the objective (probable) changes that occur in pregnancy. Enlargement of the uterus is a probable change.

Which statement is best to include when teaching a pregnant adolescent about nutritional needs of pregnancy? 1. "It is important to eat iron-rich foods like meat every day." 2. "Calcium and milk aren't needed until the third trimester." 3. "Folic acid intake is the key to having a healthy baby." 4. "You just need to pay attention to what you eat now."

Answer: 1 Explanation: 1. An inadequate iron intake is a major concern with the adolescent diet. Iron needs are high for the pregnant teen because of the requirement for iron by the enlarging maternal muscle mass and blood volume. Giving specific examples is helpful when giving nutritional information.

The nurse is explaining to a new prenatal client that the certified nurse-midwife will perform clinical pelvimetry as a part of the pelvic exam. The nurse knows that teaching has been successful when the client makes which statement about the reason for the exam? 1. "It will help us know how big a baby I can deliver vaginally." 2. "Doing this exam is a part of prenatal care at this clinic." 3. "My sister had both of her babies by cesarean." 4. "I am pregnant with my first child."

Answer: 1 Explanation: 1. By performing a series of assessments and measurements, the examiner assesses the pelvis vaginally to determine whether the size and shape are adequate for a vaginal birth; this procedure is called clinical pelvimetry.

A 43-year-old client has just had a positive pregnancy test. She cries, and states, "I just don't know what I'll do. I can't be pregnant." Which nursing diagnosis would be the most appropriate? 1. Decisional Conflict related to unexpected pregnancy 2. Knowledge, Deficient related to advanced maternal age 3. Depression related to unexpected pregnancy 4. Health Maintenance, Ineffective related to advanced maternal age

Answer: 1 Explanation: 1. Decisional Conflict related to unexpected pregnancy is the most appropriate nursing diagnosis.

The nurse is presenting a preconception counseling class. The nurse instructs the participants that niacin intake should increase during pregnancy to promote metabolic coenzyme activity. The nurse will know that teaching has been effective if a client suggests which food as a source of niacin? 1. Fish 2. Apples 3. Broccoli 4. Milk

Answer: 1 Explanation: 1. Dietary sources of niacin include meats, fish, and whole grains.

A 38-year-old client in her second trimester states a desire to begin an exercise program to decrease her fatigue. What is the most appropriate nursing response? 1. "Fatigue should resolve in the second trimester, but walking daily might help." 2. "Avoid a strenuous exercise regimen at your age. Drink coffee to combat fatigue." 3. "Avoid an exercise regimen due to your pregnancy. Try to nap daily." 4. "Fatigue will increase as pregnancy progresses, but running daily might help."

Answer: 1 Explanation: 1. Even mild to moderate exercise is beneficial during pregnancy. Regular exercise-at least 30 minutes of moderate exercise daily or at least most days of the week-is preferred.

The prenatal clinic nurse is caring for a 15-year-old client who is at 8 weeks' gestation. The client asks the nurse why she is supposed to gain so much weight. What is the best response by the nurse? 1. "Gaining 25-35 pounds is recommended for healthy fetal growth." 2. "It's what your certified nurse-midwife recommended for you." 3. "Inadequate weight gain delays lactation after delivery." 4. "Weight gain is important to ensure that you get enough vitamins."

Answer: 1 Explanation: 1. For an appropriate-weight woman, 25-35 pounds of weight gain is recommended for optimal fetal growth and development.

The nurse at the prenatal clinic has four calls to return. Which phone call should the nurse return first? 1. Client at 32 weeks, reports headache and blurred vision. 2. Client at 18 weeks, reports no fetal movement in this pregnancy. 3. Client at 16 weeks, reports increased urinary frequency. 4. Client at 40 weeks, reports sudden gush of fluid and contractions.

Answer: 1 Explanation: 1. Headache and blurred vision are signs of preeclampsia, which is potentially life-threatening for both mother and fetus. This client has top priority.

A nurse is discussing diet with a pregnant woman. Which food should the nurse advise the client to avoid during her pregnancy? 1. Bologna 2. Cantaloupe 3. Spinach 4. Cornbread

Answer: 1 Explanation: 1. Hot dogs and other luncheon meats should not be eaten during pregnancy unless they are fully cooked.

A 28-year-old woman has been an insulin-dependent diabetic for 10 years. At 36 weeks' gestation, she has an amniocentesis. A lecithin/sphingomyelin (L/S) ratio test is performed on the sample of her amniotic fluid. Because she is a diabetic, what would an obtained 2:1 ratio indicate for the fetus? 1. The fetus may or may not have immature lungs. 2. The amniotic fluid is contaminated. 3. The fetus has a neural tube defect. 4. There is blood in the amniotic fluid.

Answer: 1 Explanation: 1. Infants of diabetic mothers (IDMs) have a high incidence of false-positive results (i.e., the L/S ratio is thought to indicate lung maturity, but after birth the baby develops RDS).

The nurse is responding to phone calls. Whose call should the nurse return first? 1. A client at 37 weeks' gestation reports no fetal movement for 24 hours. 2. A client at 29 weeks' gestation reports increased fetal movement. 3. A client at 32 weeks' gestation reports decreased fetal movement X 2 days. 4. A client at 35 weeks' gestation reports decreased fetal movement X 4 hours.

Answer: 1 Explanation: 1. Lack of fetal movement can be an indication of nonreassuring fetal status or even fetal death. This client is the highest priority.

The nurse is preparing a prenatal class about infant feeding methods. The maternal nutritional requirements for breastfeeding and formula-feeding will be discussed. What statement should the nurse include? 1. "Breastfeeding requires a continued high intake of protein and calcium." 2. "Formula-feeding mothers should protect their health with a lot of calcium." 3. "Producing breast milk requires calories, but any source of food is fine." 4. "Formula-feeding mothers need a high protein intake to avoid fatigue."

Answer: 1 Explanation: 1. Lactation requires calories, along with increased protein and calcium intake.

The primiparous client has told the nurse that she is afraid she will develop hemorrhoids during pregnancy because her mother did. Which statement would be best for the nurse to make? 1. "It is not unusual for women to develop hemorrhoids during pregnancy." 2. "Most women don't have any problem until after they've delivered." 3. "If your mother had hemorrhoids, you will get them, too." 4. "If you get hemorrhoids, you probably will need surgery to get rid of them."

Answer: 1 Explanation: 1. Many pregnant women will develop hemorrhoids. Hemorrhoids are varicosities of the veins in the lower end of the rectum and anus. During pregnancy, the gravid uterus presses on the veins and interferes with venous circulation. As the pregnancy progresses, the straining that accompanies constipation can contribute to the development of hemorrhoids.

A Navajo client who is 36 weeks pregnant meets with a traditional healer as well as her physician. What does the nurse understand this to mean? 1. The client is seeking spiritual direction. 2. The client does not trust her physician. 3. The client will not adapt to mothering well. 4. The client is experiencing complications of pregnancy.

Answer: 1 Explanation: 1. Navajo clients are aware of the mind-soul connection, and might try to follow certain practices to have a healthy pregnancy and birth. Practices could include focus on peace and positive thoughts as well as certain types of prayers and ceremonies. A traditional healer may assist them.

The introduction of a new baby into the family is often the beginning of which of the following? 1. Sibling rivalry 2. Inconsistent childrearing 3. Toilet training 4. Weaning

Answer: 1 Explanation: 1. Sibling rivalry results from children's fear of change in the security of their relationships with their parents.

After teaching a pregnant client about the effects of smoking on pregnancy, the nurse knows that the client needs further education when she makes which statement? 1. "I am at increased risk for preeclampsia." 2. "I am at increased risk for preterm birth." 3. "I am at increased risk for placenta previa." 4. "I am at increased risk for abruptio placentae."

Answer: 1 Explanation: 1. Smoking is not associated with increased risk for preeclampsia.

While completing the medical and surgical history during the initial prenatal visit, the 16-year-old primigravida interrupts with "Why are you asking me all these questions? What difference does it make?" Which statement would best answer the client's questions?" 1. "We ask these questions to detect anything that happened in your past that might affect the pregnancy." 2. "We ask these questions to see whether you can have prenatal visits less often than most clients do." 3. "We ask these questions to make sure that our paperwork and records are complete and up to date." 4. "We ask these questions to look for any health problems in the past that might affect your parenting."

Answer: 1 Explanation: 1. The course of a pregnancy depends on a number of factors, including the past pregnancy history (if this is not a first pregnancy), prepregnancy health of the woman, presence of disease/illness states, family history, emotional status, and past health care.

The nurse is presenting a class to women who are currently pregnant or are planning pregnancy in the near future. Which client statement indicates that additional teaching is required? 1. "The older a woman is when she conceives, the safer the pregnancy is." 2. "Pregnant teens can have additional nutritional needs." 3. "A woman whose sisters all had hypertension will be watched carefully." 4. "Pregnancy may be more difficult to achieve in my 40s."

Answer: 1 Explanation: 1. The health risks associated with pregnancy vary by age. The risk for maternal death is significantly higher for women over age 35 and even higher for women age 40 and older. The incidence of low-birth-weight infants, preterm births, miscarriage, stillbirth, and perinatal morbidity and mortality is higher among women age 35 or older.

The kosher diet followed by many Jewish people forbids the eating of what foods? 1. Pig products and shellfish 2. Dairy products 3. All animal products 4. Dairy products and eggs

Answer: 1 Explanation: 1. The kosher diet followed by many Jewish people forbids the eating of pig products and shellfish. Certain cuts of meat from sheep and cattle are allowed, as are fish with fins and scales. In addition, many Jews believe that meat and dairy products should not be mixed or eaten at the same meal.

It is 1 week before a pregnant client's due date. The nurse notes on the chart that the client's pulse rate was 74-80 before pregnancy. Today, the client's pulse rate at rest is 90. What action should the nurse should take? 1. Chart the findings. 2. Notify the physician of tachycardia. 3. Prepare the client for an electrocardiogram (EKG). 4. Prepare the client for transport to the hospital.

Answer: 1 Explanation: 1. The pulse rate frequently increases during pregnancy, although the amount varies from almost no increase to an increase of 10 to 15 beats per minute. This is a normal response, and does not indicate a need for emergency measures or treatment.

At 32 weeks' gestation, a woman is scheduled for a second non-stress test (following one she had at 28 weeks' gestation). Which statement by the client would indicate an adequate understanding of this procedure? 1. "I can't get up and walk around during the test." 2. "I'll have an IV started before the test." 3. "I can still smoke before the test." 4. "I need to have a full bladder for this test."

Answer: 1 Explanation: 1. The purpose of the non-stress test is to determine the results of movement on fetal heart rate. The NST is typically performed with the woman in the semi-Fowler's position with a small pillow or blanket under the right hip to displace the uterus to the left.

A woman gave birth last week to a fetus at 18 weeks' gestation after her first pregnancy. She is in the clinic for follow-up, and notices that her chart states she has had one abortion. The client is upset over the use of this word. How can the nurse best explain this terminology to the client? 1. "Abortion is the obstetric term for all pregnancies that end before 20 weeks." 2. "Abortion is the word we use when someone has miscarried." 3. "Abortion is how we label babies born in the second trimester." 4. "Abortion is what we call all babies who are born dead."

Answer: 1 Explanation: 1. The term abortion means a birth that occurs before 20 weeks' gestation or the birth of a fetus-newborn who weighs less than 500 g. An abortion may occur spontaneously or it may be induced by medical or surgical means.

The nurse is seeing prenatal clients in the clinic. Which client is exhibiting expected findings? 1. 12 weeks' gestation, with fetal heart tones heard by Doppler fetoscope 2. 22 weeks' gestation, client reports no fetal movement felt yet 3. 16 weeks' gestation, fundus three finger breadths above umbilicus 4. Marked edema

Answer: 1 Explanation: 1. This is an expected finding because fetal heart tones should be heard by 12 weeks using a Doppler fetoscope.

The nurse asks a woman how her husband is dealing with the pregnancy. The nurse concludes that counseling is needed when the woman makes which statement? 1. "My husband is ready for the pregnancy to end so that we can have sex again." 2. "My husband is much more attentive to me now that I am pregnant." 3. "My husband seems more worried about our finances now than he was before the pregnancy." 4. "My husband plays his favorite music for my belly so the baby will learn to like it."

Answer: 1 Explanation: 1. This is implying that the woman and her husband are not having sex, which indicates the need for counseling. Sex is fine with a normal pregnancy.

The nurse is preparing for a postpartum home visit. The client has been home for a week, is breastfeeding, and experienced a third-degree perineal tear after vaginal delivery. The nurse should assess the client for which of the following? 1. Dietary intake of fiber and fluids 2. Dietary intake of folic acid and prenatal vitamins 3. Return of hemoglobin and hematocrit levels to baseline 4. Return of protein and albumin to predelivery levels

Answer: 1 Explanation: 1. This mother needs to avoid the risk of constipation. She might be hesitant to have a bowel movement due to anticipated pain from the perineal tear, and constipation will decrease the healing of the laceration.

The client at 14 weeks' gestation has undergone a transvaginal ultrasound to assess cervical length. The ultrasound revealed cervical funneling. How should the nurse explain these findings to the client? 1. "Your cervix has become cone-shaped and more open at the end near the baby." 2. "Your cervix is lengthened, and you will deliver your baby prematurely." 3. "Your cervix is short, and has become wider at the end that extends into the vagina." 4. "Your cervix was beginning to open but now is starting to close up again."

Answer: 1 Explanation: 1. Transvaginal ultrasound can most accurately identify shortened cervical length and cervical funneling, which is a cone-shaped indentation in the cervical os indicating cervical insufficiency or risk of preterm labor.

A pregnant client is lactose intolerant. Which alternative food could this client eat to get sufficient calcium? 1. Turnip greens 2. Green beans 3. Cantaloupe 4. Nectarines

Answer: 1 Explanation: 1. Turnip greens are rich in calcium.

The nurse evaluates the diet of a pregnant client and finds that it is low in zinc. The nurse knows that zinc intake should increase during pregnancy to promote protein metabolism. Which food should the nurse suggest in order to increase intake of zinc? 1. Shellfish 2. Bananas 3. Yogurt 4. Cabbage

Answer: 1 Explanation: 1. Zinc is found in greatest concentration in meats, shellfish, and poultry. Other good sources include whole grains and legumes.

A couple in their late 30s are pregnant for the first time. In evaluating the care delivered, the nurse assesses the mother for which of the following? Note: Credit will be given only if all correct and no incorrect choices are selected. Select all that apply. 1. Makes appropriate healthcare choices 2. Receives effective healthcare through the pregnancy 3. Has a partner who is not interested in child care 4. Cannot cope with her life change 5. Wishes to have amniocentesis done

Answer: 1, 2 Explanation: 1. Expected outcomes of nursing care for a pregnant couple over 35 include that the client and her partner are knowledgeable about the pregnancy and make appropriate healthcare choices. 2. Expected outcomes of nursing care for a pregnant couple over 35 include the client and her partner receive effective health care throughout the pregnancy, birth, and postpartum period.

A client at 32 weeks' gestation comes to the clinic with urinary burning and frequency. The nurse explains that urinary tract infections are common in pregnancy due to which of the following? Note: Credit will be given only if all correct and no incorrect choices are selected. Select all that apply. 1. Ureteral atonia 2. Stasis of urine 3. Increased glomerular filtration rate 4. Increased renal plasma flow 5. Increased clearance of urea

Answer: 1, 2 Explanation: 1. The presence of amino acids and glucose in the urine in conjunction with the tendency toward ureteral atonia and stasis of urine in the ureters may increase the risk of urinary tract infection. 2. The presence of amino acids and glucose in the urine in conjunction with the tendency toward ureteral atonia and stasis of urine in the ureters may increase the risk of urinary tract infection.

During the initial prenatal visit, the nurse obtains a weight of 42 kg (92.4 lb). The nurse must further assess the client for information about which of the following? Note: Credit will be given only if all correct and no incorrect choices are selected. Select all that apply. 1. Eating habits 2. Foods regularly eaten 3. Income limitations 4. Blood pressure and pulse rate 5. Weight loss during pregnancy

Answer: 1, 2, 3 Explanation: 1. For a client whose weight is less than 100 lb, the nurse would obtain information on eating habits. 2. For a client whose weight is less than 100 lb, the nurse would obtain information on foods regularly eaten. 3. For a client whose weight is less than 100 lb, the nurse would obtain information on income limitations.

The OB-GYN nurse works in a clinic with a culturally diverse group of clients whose specific actions during pregnancy are often determined by cultural beliefs. The nurse recognizes that these beliefs about pregnancy and childbirth fall into which categories? Note: Credit will be given only if all correct and no incorrect choices are selected. Select all that apply. 1. Prescriptive beliefs 2. Restrictive beliefs 3. Taboos 4. Cultural humility 5. Folk treatment beliefs

Answer: 1, 2, 3 Explanation: 1. Prescriptive beliefs or requirements describe expected behaviors. 2. Restrictive beliefs are stated negatively and limit behaviors. 3. Taboo beliefs refer to specific supernatural consequences.

The clinic nurse is culturally sensitive when, while assessing the pregnant client, he asks about which of the following? Note: Credit will be given only if all correct and no incorrect choices are selected. Select all that apply. 1. The family's expectations of the healthcare system 2. Which cultural practices should be incorporated into care 3. Any alternative healer who should be consulted 4. Positive consequences of the client's healthcare beliefs 5. The client's giving up her practices and adopting the practices of the dominant culture

Answer: 1, 2, 3 Explanation: 1. The nurse needs to ask about the client's expectations of the healthcare system. 2. The nurse should ask about any cultural or spiritual practices that should be incorporated into care. 3. The culturally sensitive nurse will ask whether any alternative healer should be consulted about care.

What self-care measures would a nurse recommend for a client in her first trimester to reduce the discomfort of nausea and vomiting? Note: Credit will be given only if all correct and no incorrect choices are selected. Select all that apply. 1. Avoid odors or causative factors. 2. Have small but frequent meals. 3. Drink carbonated beverages. 4. Drink milk before arising in the morning. 5. Eat highly seasoned food.

Answer: 1, 2, 3 Explanation: 1. The nurse would recommend for a client in her first trimester to avoid odors and caustic factors to reduce the discomfort of nausea and vomiting. 2. The nurse would recommend for a client in her first trimester to have small but frequent meals to reduce the discomfort of nausea and vomiting. 3. The nurse would recommend for a client in her first trimester to drink carbonated beverages to reduce the discomfort of nausea and vomiting.

The prenatal period should be used to expose the prospective parents to up-to-date, evidence-based information about which of the following topics? Note: Credit will be given only if all correct and no incorrect choices are selected. Select all that apply. 1. Breastfeeding 2. Pain relief 3. Obstetric complications and procedures 4. Toddler care 5. Antepartum adjustment

Answer: 1, 2, 3 Explanation: 1. The prenatal period should expose prospective parents to up-to-date, evidence-based information about breastfeeding. 2. The prenatal period should expose prospective parents to up-to-date, evidence-based information about pain relief. 3. The prenatal period should expose prospective parents to up-to-date, evidence-based information about obstetric complications and procedures.

The partner of a pregnant client comes to the clinic with her. He complains to the nurse that he is experiencing different physical changes. The nurse determines he is experiencing couvade when he describes which symptoms? Note: Credit will be given only if all correct and no incorrect choices are selected. Select all that apply. 1. Fatigue 2. Increased appetite 3. Headache 4. Backache 5. High anxiety level

Answer: 1, 2, 3, 4 Explanation: 1. Couvade is demonstrated by increased fatigue in the partner. 2. Couvade is demonstrated by an increased appetite in the partner. 3. Couvade is demonstrated by the partner's having headaches. 4. Couvade is demonstrated by the partner's experiencing backache.

During home care of a low-risk pregnant client, the nurse provides care by assessing which of the following? Note: Credit will be given only if all correct and no incorrect choices are selected. Select all that apply. 1. Urine 2. Weight 3. Diet 4. Pelvic measurements 5. Physical activity

Answer: 1, 2, 3, 5 Explanation: 1. Assessment of the client's urine can be done in the home setting. 2. Obtaining the client's weight can be done in the home setting. 3. Assessing the client's dietary intake can be done in the home setting. 5. The client's physical activity can be assessed in the home setting.

The nurse is counseling a group of first-trimester clients on diet increases that are necessary during pregnancy. Which information would be necessary to tell the pregnant women? Note: Credit will be given only if all correct choices and no incorrect choices are selected. Select all that apply. 1. An increase of protein is necessary to provide amino acids necessary for fetal development. 2. Protein contributes to the body's overall energy metabolism. 3. The recommended protein during pregnancy is 70 grams each day. 4. The increased amount of protein that a pregnant woman needs is 15 grams a day. 5. The quality of protein is as important as the amount.

Answer: 1, 2, 3, 5 Explanation: 1. During pregnancy, the woman needs increased amounts of protein to provide amino acids for fetal development, blood volume expansion, and growth of other maternal tissues. 2. Protein contributes to overall energy metabolism. 3. The amount of protein recommended each day during pregnancy is 70 grams. 5. Plant proteins can meet a woman's protein needs; however, more of a given protein may be needed because animal proteins are of higher quality.

Which of the following are specific culturally sensitive nursing considerations the nurse integrates into care of the pregnant client? Note: Credit will be given only if all correct and no incorrect choices are selected. Select all that apply. 1. Counseling about home remedies 2. Discussing components of a balanced diet 3. Refusing to discuss birthing choices 4. Encouraging use of support systems 5. Instructing the client to use no home remedies

Answer: 1, 2, 4 Explanation: 1. The culturally sensitive nurse should find out what medications and home remedies the client is using, and counsel the client regarding overall effects. 2. The culturally sensitive nurse will discuss the importance of a well-balanced diet during pregnancy with consideration of the client's cultural beliefs and practices. 4. Encouraging the use of support systems and spiritual aids that provide comfort for the mother is important in culturally sensitive care.

The nurse is planning an early-pregnancy class session on nutrition. Which information should the nurse include? Note: Credit will be given only if all correct choices and no incorrect choices are selected. Select all that apply. 1. Protein is important for fetal development. 2. Iron helps both mother and baby maintain the oxygen-carrying capacity of the blood. 3. Calcium prevents constipation at the end of pregnancy. 4. Zinc facilitates synthesis of RNA and DNA. 5. Vitamin A promotes development of the baby's eyes.

Answer: 1, 2, 4, 5 Explanation: 1. During pregnancy, the woman needs increased amounts of protein to provide amino acids for fetal development. 2. Iron deficiency anemia is associated with an increased incidence of preterm birth, low-birth-weight infants, and maternal and infant mortality. 4. Zinc is involved in RNA and DNA synthesis, and milk production during lactation. 5. Vitamin A promotes healthy formation and development of the fetal eyes.

Remedies for back pain in pregnancy that are supported by research evidence and may safely be taught to any pregnant woman by the nurse include which of the following? Note: Credit will be given only if all correct choices and no incorrect choices are selected. Select all that apply. 1. Pelvic tilt 2. Water aerobics 3. Sit-ups 4. Proper body mechanics 5. Maintaining good posture

Answer: 1, 2, 4, 5 Explanation: 1. The pelvic tilt can help restore proper body alignment and relieve back pain. 2. Exercise is an effective treatment for lower back pain. Exercise in water seems to provide benefits while being physically comfortable for expectant mothers. 4. The use of proper posture and good body mechanics throughout pregnancy is important. 5. Good posture is important because it allows more room for the stomach to function.

A pregnant client calls the clinic nurse to say she is worried about symptoms she is experiencing. The nurse advises the client to come immediately to the clinic because of which reported symptoms? Note: Credit will be given only if all correct and no incorrect choices are selected. Select all that apply. 1. Vaginal bleeding 2. Abdominal pain 3. Constipation 4. Epigastric pain 5. Blurring of vision

Answer: 1, 2, 4, 5 Explanation: 1. Vaginal bleeding can indicate abruptio placentae, placenta previa, or lesions of cervix or vagina, or it can be "bloody show," and requires that the client be seen. 2. Abdominal pain can signal premature labor or abruptio placentae, and requires that the client be seen. 4. Epigastric pain must be evaluated, as it can indicate preeclampsia or ischemia in a major abdominal vessel. 5. Dizziness, blurring of vision, double vision, or spots before the eyes can indicate either hypertension and/or preeclampsia and requires the client be seen.

) The client and her partner are carriers of sickle-cell disease. They are considering prenatal diagnosis with either amniocentesis or chorionic villus sampling (CVS). Which statements indicate that further teaching is needed on these two diagnostic procedures? Note: Credit will be given only if all correct choices and no incorrect choices are selected. Select all that apply. 1. "Chorionic villus sampling carries a lower risk of miscarriage." 2. "Amniocentesis can be done earlier in my pregnancy than CVS." 3. "Neither test will conclusively diagnose sickle-cell disease in our baby." 4. "The diagnosis comes sooner if we have CVS, not amniocentesis." 5. "Amniocentesis is more accurate in diagnosis than the CVS."

Answer: 1, 2, 5 Explanation: 1. CVS has a risk of spontaneous abortion of 0.3% in cases. This rate is higher than second trimester amniocentesis. 2. CVS is performed in some medical centers for first trimester diagnosis after 9 completed weeks. Amniocentesis is performed between 15 and 20 weeks' gestation. 5. Much like amniocentesis, chorionic villus sampling (CVS) is a procedure that is used to detect genetic, metabolic, and DNA abnormalities. CVS permits earlier diagnosis than can be obtained by amniocentesis.

The pregnant client states she does not want "to take all these supplements." What recommendations could the nurse make for the client? Note: Credit will be given only if all correct choices and no incorrect choices are selected. Select all that apply. 1. "Folic acid has been found to be essential for minimizing the risk of neural tube defects." 2. "You do not have to take these supplements if you think you are healthy enough." 3. "Most women do not have adequate intake of iron pre-pregnancy, and iron needs increase with pregnancy." 4. "These medications do the same thing. I will call your physician to cancel one of your medications." 5. "You should take the folic acid, but the vitamins are not that important."

Answer: 1, 3 Explanation: 1. An inadequate intake of folic acid has been associated with neural tube defects (NTDs) (e.g., spina bifida, anencephaly). 3. Iron is essential because many pregnant women do not have adequate intake of iron before pregnancy.

During a client's initial prenatal visit, the nurse must assess and document the client's current medical history, including which information? Note: Credit will be given only if all correct and no incorrect choices are selected. Select all that apply. 1. Body mass index 2. Infections before the last menstrual period 3. Homeopathic or herbal medication use 4. Blood type and Rh factor 5. History of previous pregnancies

Answer: 1, 3, 4 Explanation: 1. The body mass index is an important part of the current medical history to be assessed and documented. 3. Homeopathic and herbal medication use is important for the nurse to assess and document in the current medical history. 4. The blood type must be assessed and documented in the current medical history, as must the Rh factor.

A pregnant client at 30 weeks' gestation has had a steady rise in blood pressure. She is now 20 mmHg above her systolic baseline. The nurse advises her to immediately report which symptoms? Note: Credit will be given only if all correct and no incorrect choices are selected. Select all that apply. 1. Dizziness 2. Even a small amount of dependent edema 3. Spots before her eyes 4. Persistent nausea and vomiting 5. Vaginal spotting

Answer: 1, 3 Explanation: 1. Dizziness can be a sign of hypertension or preeclampsia, and should be reported immediately. 3. Spots before the eyes can be a sign of hypertension or preeclampsia, and should be reported immediately.

The nurse is instructing a pregnant client on her nutritional needs. The nurse tells the client that nutrition is needed for fetal development because fetal growth occurs in overlapping stages from increases of which of the following? Note: Credit will be given only if all correct and no incorrect choices are selected. Select all that apply. 1. Cell numbers 2. Cell membranes 3. Cell size and number 4. Cell size alone 5. Vitamin and mineral intake

Answer: 1, 3, 4 Explanation: 1. Fetal growth occurs by an increase in cell numbers in one stage. 3. Fetal growth occurs by an increase in cell size and number during one stage. 4. Fetal growth occurs by an increase in cell size alone during one stage.

Postpartum nutritional status is determined primarily by assessing which of the following? Note: Credit will be given only if all correct choices and no incorrect choices are selected. Select all that apply. 1. Dietary history 2. Menstrual history 3. Mother's weight 4. Hemoglobin levels 5. Mother's height

Answer: 1, 3, 4 Explanation: 1. Postpartum nutritional status is determined by assessing the new mother's dietary history. 3. Postpartum nutritional status is determined by assessing the new mother's weight. 4. Postpartum nutritional status is determined by assessing the new mother's hemoglobin levels.

The nurse in the OB-GYN clinic is working with a client who is seeking her initial prenatal visit. The nurse will use the acronym TPAL to document the client's number of which of the following? Note: Credit will be given only if all correct and no incorrect choices are selected. Select all that apply. 1. Term infants born 2. Children living in the home 3. Pregnancies ending in abortion 4. Preterm infants born 5. Pregnancies that occurred

Answer: 1, 3, 4 Explanation: 1. T: number of term births the woman has experienced 3. A: number of pregnancies ending in either spontaneous or therapeutic abortion 4. P: number of preterm births

The nurse is assessing a client in the third trimester of pregnancy. What physiologic changes in the client are expected? Note: Credit will be given only if all correct choices and no incorrect choices are selected. Select all that apply. 1. The client's chest circumference has increased by 6 cm during the pregnancy. 2. The client has a narrowed subcostal angle. 3. The client is using thoracic breathing. 4. The client may have epistaxis. 5. The client has a productive cough.

Answer: 1, 3, 4 Explanation: 1. The chest increase compensates for the elevated diaphragm. 3. Breathing changes from abdominal to thoracic as pregnancy progresses. 4. Epistaxis (nosebleeds) may occur and are primarily the result of estrogen-induced edema and vascular congestion of the nasal mucosa.

The nurse is explaining clinical pelvimetry to a client. The nurse explains that the anteroposterior diameters consist of which of the following? Note: Credit will be given only if all correct and no incorrect choices are selected. Select all that apply. 1. Diagonal conjugate 2. Transverse diameter 3. Conjugata vera 4. Obstetric conjugate 5. Oblique diameter

Answer: 1, 3, 4 Explanation: 1. The diagonal conjugate is a part of the anteroposterior diameter measurement. 3. The conjugata vera is a part of the anteroposterior diameter measurement. 4. The obstetric conjugate is a part of the anteroposterior diameter measurement.

The nurse understands that a client's pregnancy is progressing normally when what physiologic changes are documented on the prenatal record of a woman at 36 weeks' gestation? Note: Credit will be given only if all correct choices and no incorrect choices are selected. Select all that apply. 1. The joints of the pelvis have relaxed, causing a waddling gait. 2. The cervix is firm and blue-purple in color. 3. The uterus vasculature contains one sixth of the total maternal blood volume. 4. Gastric emptying time is delayed, and the client complains of constipation and bloating. 5. Supine hypotension occurs when the client lies on her back.

Answer: 1, 3, 4, 5 Explanation: 1. The sacroiliac, sacrococcygeal, and pubic joints of the pelvis relax in the later part of the pregnancy, presumably as a result of hormonal changes. This often causes a waddling gait. 3. By the end of pregnancy, one sixth of the total maternal blood volume is contained within the vascular system of the uterus. 4. Gastric emptying time and intestinal motility are delayed, leading to frequent complaints of bloating and constipation, which can be aggravated by the smooth muscle relaxation and increased electrolyte and water reabsorption in the large intestine. 5. The enlarging uterus may exert pressure on the vena cava when the woman lies supine, causing a drop in blood pressure. This is called the vena caval syndrome, or supine hypotension.

A client at 34 weeks' gestation complains about pyrosis. The nurse teaches the patient that approaches to relieve the pyrosis include which of the following? Note: Credit will be given only if all correct and no incorrect choices are selected. Select all that apply. 1. Eat small, frequent meals 2. Use high-sodium antacids 3. Avoid fried, fatty foods 4. Take sodium bicarbonate after meals 5. Do not lie down after eating

Answer: 1, 3, 5 Explanation: 1. Pyrosis (heartburn) can be relieved by eating small, more frequent meals. 3. Avoiding fatty, fried foods can relieve pyrosis. 5. Sitting up after meals will help decrease the pyrosis.

Women with eating disorders who become pregnant are at risk for a variety of complications including which of the following? Note: Credit will be given only if all correct choices and no incorrect choices are selected. Select all that apply. 1. Premature birth 2. Too many nutrients available for the fetus 3. Miscarriage 4. High birth weight 5. Perinatal mortality

Answer: 1, 3, 5 Explanation: 1. Risks to the mother and baby include premature birth. 3. Risks to the mother and baby include miscarriage. 5. Risks to the mother and baby include perinatal mortality.

The clinic nurse is assessing how the prenatal client is meeting developmental tasks using Rubin's tasks, including which of the following? Note: Credit will be given only if all correct and no incorrect choices are selected. Select all that apply. 1. Ensuring safe passage through pregnancy, labor, and birth. 2. Turning in on oneself to focus on the child. 3. Seeking commitment and acceptance of self as mother to the infant. 4. Completing the tasks of nesting at the appropriate time. 5. Seeking acceptance of the child by others.

Answer: 1, 3, 5 Explanation: 1. The tasks Rubin identified form the basis for a mutually gratifying relationship with the baby, and include ensuring safe passage through pregnancy, labor, and birth. 3. The tasks Rubin identified form the basis for a mutually gratifying relationship with the baby, and include seeking commitment and acceptance of self as mother. 5. The tasks Rubin identified form the basis for a mutually gratifying relationship with the baby, and include seeking acceptance of the child by others.

To answer a client's question about home pregnancy tests and their accuracy, the nurse must know that accuracy is affected by which of the following? Note: Credit will be given only if all correct and no incorrect choices are selected. Select all that apply. 1. Unclear directions 2. Unable to comprehend the directions 3. Blood in the specimen giving a false reading 4. Completing the test too late 5. Tagged antibodies becoming outdated

Answer: 1, 4 Explanation: 1. Women may not comprehend the HPT instructions, which can affect the accuracy results. 4. False-negative results typically occur when the test is completed too early or too late.

The nurse is preparing a class for expectant fathers. Which information should the nurse include? 1. Siblings adjust readily to the new baby. 2. Sexual activity is safe for normal pregnancy. 3. The expectant mother decides the feeding method. 4. Fathers are expected to be involved in labor and birth.

Answer: 2 Explanation: 2. In a healthy pregnancy, there is no medical reason to limit sexual activity.

The pregnant teen who was prescribed prenatal vitamins at her initial prenatal visit states that she does not like to take them. How should the nurse respond? Note: Credit will be given only if all correct choices and no incorrect choices are selected. Select all that apply. 1. "Folic acid has been found to be essential for minimizing the risk of neural tube defects." 2. "You do not have to take these supplements if you think you are healthy enough." 3. "These medications do the same thing. I will call your doctor to cancel one of your medications." 4. "You can trust your doctor to know what you need." 5. "You need the supplements because your dietary intake may not be adequate for fetal development."

Answer: 1, 5 Explanation: 1. The CDC estimates that most neural tube defects could be prevented if women followed folic acid supplementation recommendations before they know they are pregnant. 5. One role of the nurse is educator, and this client needs additional information on why she needs the supplements. This response answers the client's concerns.

The nurse is reviewing amniocentesis results. Which of the following would indicate that client care was appropriate? 1. The client who is Rh-positive received Rh immune globulin after the amniocentesis. 2. The client was monitored for 30 minutes after completion of the test. 3. The client began vaginal spotting before leaving for home after the test. 4. The client identified that she takes insulin before each meal and at bedtime.

Answer: 2 Explanation: 2. 20 to 30 minutes of fetal monitoring is performed after the amniocentesis.

A nurse examining a prenatal client recognizes that a lag in progression of measurements of fundal height from week to week and month to month could signal what condition? 1. Twin pregnancy 2. Intrauterine growth restriction 3. Hydramnios 4. Breech position

Answer: 2 Explanation: 2. A lag in progression of measurements of fundal height from month to month could signal intrauterine growth restriction (IUGR).

A client at 37 weeks' gestation has a mildly elevated blood pressure. Her antenatal testing demonstrates three contractions in 10 minutes, no decelerations, and accelerations four times in 1 hour. What would this test be considered? 1. Positive non-stress test 2. Negative contraction stress test 3. Positive contraction stress test 4. Negative non-stress test

Answer: 2 Explanation: 2. A negative CST shows three contractions of good quality lasting 40 or more seconds in 10 minutes without evidence of late decelerations. This is the desired result.

The pregnant client at 14 weeks' gestation is in the clinic for a regular prenatal visit. Her mother also is present. The grandmother-to-be states that she is quite uncertain about how she can be a good grandmother to this baby because she works full-time. Her own grandmother was retired, and was always available when needed by a grandchild. What is the nurse's best response to this concern? 1. "Don't worry. You'll be a wonderful grandmother. It will all work out fine." 2. "What are your thoughts on what your role as grandmother will include?" 3. "As long as there is another grandmother available, you don't have to worry." 4. "Grandmothers are supposed to be available. You should retire from your job."

Answer: 2 Explanation: 2. Although relationships with parents can be very complex, the expectant grandparents often become increasingly supportive of the expectant couple, even if conflicts previously existed. But it can be difficult for even sensitive grandparents to know how deeply to become involved in the childrearing process. In some areas, classes for grandparents provide information about changes in birthing and parenting practices.

The breastfeeding mother is concerned that her milk production has decreased. The nurse knows that further client teaching is needed based on which statement? 1. "I am drinking a minimum of 8 to 10 glasses of liquid a day." 2. "I have started cutting back on my protein intake." 3. "At least three times a day, I drink a glass of milk." 4. "My calorie intake is higher than during the pregnancy."

Answer: 2 Explanation: 2. An adequate protein intake is essential while breastfeeding because protein is an important component of breast milk.

A 25-year-old primigravida is at 20 weeks' gestation. The nurse takes her vital signs and notifies the healthcare provider immediately because of which finding? 1. Pulse 88/minute 2. Rhonchi in both bases 3. Temperature 37.4° C (99.3° F) 4. Blood pressure 122/78

Answer: 2 Explanation: 2. Any abnormal breath sounds should be reported to the healthcare provider.

What would the nurse do to accurately assess a pregnant client's food intake? 1. Assess her most recent laboratory values. 2. Ask her to complete a nutritional questionnaire. 3. Observe for signs of hunger. 4. Ask about her cooking facilities.

Answer: 2 Explanation: 2. Diet may be evaluated using a food frequency questionnaire, which lists common categories of foods and asks the woman how frequently in a day (or week) she consumes foods from the list.

Which of the following is important for the development of the central nervous system of the fetus? 1. Calcium and phosphorus 2. Essential fatty acids 3. Iron 4. Vitamin D

Answer: 2 Explanation: 2. Essential fatty acids are important for the development of the central nervous system of the fetus. Of particular interest are the omega-3 fatty acids and their derivatives.

The nurse receives a phone call from a client who claims she is pregnant. The client reports that she has regular menses that occur every 28 days and last 5 days. The first day of her last menses was April 10. What would the client's estimated date of delivery (EDD) be if she is pregnant? 1. Nov. 13 2. Jan. 17 3. Jan. 10 4. Dec. 3

Answer: 2 Explanation: 2. The due date is Jan. 17. Nagele's rule is to add 7 days to the last menstrual period and subtract 3 months. The last menstrual period is April 10, therefore Jan. 17 is the EDD.

A client who is experiencing her first pregnancy has just completed the initial prenatal examination with a certified nurse-midwife. Which statement indicates that the client needs additional information? 1. "Because we heard the baby's heartbeat, I am undoubtedly pregnant." 2. "Because I haven't felt the baby move yet, we don't know whether I'm pregnant." 3. "My last period was 2 months ago, which means I'm 2 months along." 4. "The increased size of my uterus means that I am finally pregnant."

Answer: 2 Explanation: 2. Fetal movement is a subjective, or presumptive, change of pregnancy, and is not a reliable indicator in the early months of pregnancy.

A client at 16 weeks' gestation has a hematocrit of 35%. Her prepregnancy hematocrit was 40%. Which statement by the nurse best explains this change? 1. "Because of your pregnancy, you're not making enough red blood cells." 2. "Because your blood volume has increased, your hematocrit count is lower." 3. "This change could indicate a serious problem that might harm your baby." 4. "You're not eating enough iron-rich foods like meat."

Answer: 2 Explanation: 2. Hemoglobin and hematocrit levels drop in early to mid-pregnancy as a result of pregnancy-associated hemodilution. Because the plasma volume increase (50%) is greater than the erythrocyte increase (25%), the hematocrit decreases slightly.

Carbohydrates provide the body's primary source of energy as well as fiber necessary for proper bowel functioning. If the carbohydrate intake is not adequate, the body will use which of the following for energy? 1. Iron 2. Protein 3. Vitamin C 4. Vitamin D

Answer: 2 Explanation: 2. If the carbohydrate intake is not adequate, the body uses protein for energy. Protein then becomes unavailable for growth needs.

A Chinese woman who is 12 weeks pregnant reports to the nurse that ginseng and bamboo leaves help reduce her anxiety. How should the nurse respond to this client? 1. Advise the client to give up the bamboo leaves but to continue taking ginseng. 2. Advise the client to give up all herbal remedies. 3. Tell the client that her remedies have no scientific foundation. 4. Assess where the client obtains her remedy, and investigate the source.

Answer: 2 Explanation: 2. In some cases, the nurse might want to suggest remedies that may be more effective than herbal remedies. However, if the home remedy is not harmful, there is no reason for the nurse to ask a client to discontinue this practice.

A client presents to the antepartum clinic with a history of a 20-pound weight loss. Her pregnancy test is positive. She is concerned about gaining the weight back, and asks the nurse if she can remain on her diet. What is the nurse's best response? 1. "As long as you supplement your diet with the prenatal vitamin, the amount of weight you gain in pregnancy is not significant." 2. "I understand that gaining weight after such an accomplishment might not look attractive, but weight gain during pregnancy is important for proper fetal growth." 3. "Dieting during pregnancy is considered child neglect." 4. "Excessive weight gain in pregnancy is due to water retention, so weight loss following birth will not be an issue."

Answer: 2 Explanation: 2. Maternal weight gain is an important factor in fetal growth and in infant birth weight. An adequate weight gain over time indicates an adequate caloric intake.

A pregnant client who swims 3-5 times per week asks the nurse whether she should stop this activity. What is the appropriate nursing response? 1. "You should decrease the number of times you swim per week." 2. "Continuing your exercise program would be beneficial." 3. "You should discontinue your exercise program immediately." 4. "You should consider a less strenuous type of exercise."

Answer: 2 Explanation: 2. Mild to moderate exercise is beneficial during pregnancy. Regular exercise-at least 30 minutes of moderate exercise daily or at least most days of the week-is preferred.

The clinic nurse is assisting with an initial prenatal assessment. The following findings are present: spider nevi present on lower legs; dark pink, edematous nasal mucosa; mild enlargement of the thyroid gland; mottled skin and pallor on palms and nail beds; heart rate 88 with murmur present. What is the best action for the nurse to take based on these findings? 1. Document the findings on the prenatal chart. 2. Have the physician see the client today. 3. Instruct the client to avoid direct sunlight. 4. Analyze previous thyroid hormone lab results.

Answer: 2 Explanation: 2. Mottling of the skin is indicative of possible anemia. These abnormalities must be reported to the healthcare provider immediately.

The nurse is assessing a primiparous client who indicates that her religion is Judaism. Why is this information is pertinent for the nurse to assess? 1. Religious and cultural background can impact what a client eats during pregnancy. 2. It provides a baseline from which to ask questions about the client's religious and cultural background. 3. Knowing the client's beliefs and behaviors regarding pregnancy is not important. 4. Clients sometimes encounter problems in their pregnancies based on what religion they practice.

Answer: 2 Explanation: 2. Nurses have an obligation to be aware of other cultures and develop a culturally sensitive plan of care to meet the needs of the childbearing woman and her family.

A woman confides to the nurse that she has pica. What alternative could the nurse suggest to the client? 1. Replace laundry starch with salt. 2. Replace ice with frozen fruit pops. 3. Replace soap with cream cheese. 4. Replace soil with nuts.

Answer: 2 Explanation: 2. Some women are able to switch to frozen fruit pops instead of ice.

A client who is in the second trimester of pregnancy tells the nurse that she has developed a darkening of the line in the midline of her abdomen from the symphysis pubis to the umbilicus. What other expected changes during pregnancy might she also notice? 1. Lightening of the nipples and areolas 2. Reddish streaks called striae on her abdomen 3. A decrease in hair thickness 4. Small purplish dots on her face and arms

Answer: 2 Explanation: 2. Striae, or stretch marks, are reddish, wavy, depressed streaks that may occur over the abdomen, breasts, and thighs as pregnancy progresses.

The nurse begins a prenatal assessment on a 25-year-old primigravida at 20 weeks' gestation and immediately contacts the healthcare provider because of which finding? 1. Pulse 88/minute 2. Respirations 30/minute 3. Temperature 37.4° C (99.3° F) 4. Blood pressure 118/82

Answer: 2 Explanation: 2. Tachypnea is not a normal finding and requires medical care.

The nurse anticipates that the physician will most likely order a cervicovaginal fetal fibronectin test for which client? 1. The client at 34 weeks' gestation with gestational diabetes 2. The client at 32 weeks' gestation with regular uterine contractions 3. The client at 37 weeks' multi-fetal gestation 4. The client at 20 weeks' gestation with ruptured amniotic membranes

Answer: 2 Explanation: 2. The absence of cervicovaginal fFN between 20 and 34 weeks' gestation has been shown to be a strong predictor of a woman not experiencing preterm birth due to spontaneous preterm labor or premature rupture of membranes. Positive findings indicate a 99% probability of birth within the next 2 weeks.

Of all the clients who have been scheduled to have a biophysical profile, the nurse should check with the physician and clarify the order for which client? 1. A gravida with intrauterine growth restriction 2. A gravida with mild hypotension of pregnancy 3. A gravida who is postterm 4. A gravida who complains of decreased fetal movement for 2 days

Answer: 2 Explanation: 2. The biophysical profile is used when there is a risk of placental and/or fetal compromise. The gravida with mild hypotension will need to be monitored more closely throughout the pregnancy, but is not a candidate at present for a biophysical profile.

On examination of the prenatal client, the nurse is aware that she will assess for a bluish pigmentation of the vagina. What is this objective (probable) sign of pregnancy also known as? 1. Hegar sign 2. Chadwick sign 3. Nightingale sign 4. Goodell sign

Answer: 2 Explanation: 2. The blue-purple discoloration of the cervix is Chadwick sign.

A 16-year-old pregnant client is seen at her 10-weeks'-gestation visit. She tells the nurse that she felt the baby move that morning. What response by the nurse is appropriate? 1. "That is very exciting. The baby must be very healthy." 2. "Would you please describe what you felt for me?" 3. "That is impossible. The baby is not big enough yet." 4. "Would you please let me see whether I can feel the baby?"

Answer: 2 Explanation: 2. The nurse should ask the client to describe what she felt, as 10 weeks' gestation is too early to feel fetal movement.

Nurses who are interacting with expectant families from a different culture or ethnic group can provide more effective, culturally sensitive nursing care by doing what? 1. Recognizing that ultimately it is the family's right to make a woman's healthcare choices. 2. Obtaining a medical interpreter of the language the client speaks. 3. Evaluating whether the client's healthcare beliefs have any positive consequences for her health. 4. Accepting personal biases, attitudes, stereotypes, and prejudices.

Answer: 2 Explanation: 2. The nurse should provide for the services of an interpreter if language barriers exist

The pregnant client has asked the nurse what kinds of medications cause birth defects. Which statement would best answer this question? 1. "Birth defects are very rare. Don't worry; your doctor will watch for problems." 2. "To be safe, don't take any medication without talking to your doctor." 3. "Too much vitamin C is one of the most common issues." 4. "Almost all medications will cause birth defects in the first trimester."

Answer: 2 Explanation: 2. The nurse should remind the client of the need to check with her caregiver about medications. If a woman has taken a drug in category D or X, she should be informed of the risks associated with that drug and of her alternatives.

A prenatal educator is asking a partner about normal psychological adjustment of an expectant mother during the second trimester of pregnancy. Which answer by the partner would indicate a typical expectant mother's response to pregnancy? 1. "She is very body-conscious, and hates every little change." 2. "She daydreams about what kind of parent she is going to be." 3. "I haven't noticed anything. I just found out she was pregnant." 4. "She has been having dreams at night about misplacing the baby."

Answer: 2 Explanation: 2. The second trimester brings increased introspection and consideration of how she will parent. She might begin to get furniture and clothing as concrete preparation, and feels movement and is aware of the fetus and incorporates it into herself.

The adolescent client reports to the clinic nurse that her period is late, but that her home pregnancy test is negative. What should the nurse explain that these findings most likely indicate? 1. "This means you are not pregnant." 2. "You might be pregnant, but it might be too early for your home test to be accurate." 3. "We don't trust home tests. Come to the clinic for a blood test." 4. "Most people don't use the tests correctly. Did you read the instructions?"

Answer: 2 Explanation: 2. This is a true statement. Most home pregnancy tests have low false-positive rates, but the false-negative rate is slightly higher. Repeating the test in a week is recommended.

The prenatal clinic nurse is designing a new prenatal intake information form for pregnant clients. Which question is best to include on this form? 1. Where was the father of the baby born? 2. Do genetic diseases run in the family of the baby's father? 3. What is the name of the baby's father? 4. Are you married to the father of the baby?

Answer: 2 Explanation: 2. This question has the highest priority because it gets at the physiologic issue of inheritable genetic diseases that might directly impact the baby.

The school nurse is planning a class about nutrition for pregnant teens, several of whom have been diagnosed with iron-deficiency anemia. In order to increase iron absorption, the nurse would encourage the teens to consume more of what beverage? 1. Gatorade 2. Orange juice 3. Milk 4. Green tea

Answer: 2 Explanation: 2. Vitamin C is found in citrus fruits and juices, and is known to enhance the absorption of iron from meat and non-meat sources.

A pregnant teenage client is diagnosed with iron-deficiency anemia. Which nutrient should the nurse encourage her to take to increase iron absorption? 1. Vitamin A 2. Vitamin C 3. Vitamin D 4. Vitamin E

Answer: 2 Explanation: 2. Vitamin C is known to enhance the absorption of iron from meat and nonmeat sources.

The client is at 6 weeks' gestation, and is spotting. The client had an ectopic pregnancy 1 year ago, so the nurse anticipates that the physician will order which intervention? 1. A urine pregnancy test 2. The client to be seen next week for a full examination 3. An antiserumpregnancy test 4. An ultrasound to be done

Answer: 3 Explanation: 3. A β-Subunit radioimmunoassay (RIA) uses an antiserum with specificity for the β-subunit of hCG in blood plasma. This test may not only detect pregnancy but also detect an ectopic pregnancy or trophoblastic disease.

The nurse recognizes that subjective pregnancy changes such as amenorrhea can be caused by which conditions? Note: Credit will be given only if all correct and no incorrect choices are selected. Select all that apply. 1. Goodell sign 2. Anemia 3. Pseudocyesis 4. Thyroid dysfunction 5. Fetal heartbeat

Answer: 2, 3, 4 Explanation: 2. Anemia can cause amenorrhea, and is a subjective sign of pregnancy. 3. Pseudocyesis (intense desire for pregnancy) can cause amenorrhea. 4. Thyroid dysfunction can cause amenorrhea, and is a subjective sign of pregnancy.

The nurse is seeing a client who asks about the accuracy of Nagele's rule. The nurse explains that accuracy can be compromised under which conditions? Note: Credit will be given only if all correct and no incorrect choices are selected. Select all that apply. 1. There is a history of regular menses every 28 days. 2. Amenorrhea is present and ovulation occurs with breastfeeding. 3. Oral contraception was discontinued, but no regular menstruation was established. 4. There has been 1 or more months of amenorrhea. 5. There is an accurate date for the last menstrual period.

Answer: 2, 3, 4 Explanation: 2. Nagele's rule is not always accurate for women who have amenorrhea but are ovulating and conceive while breastfeeding. 3. Nagele's rule is not always accurate for women who conceive before regular menstruation is established following discontinuation of oral contraceptives or termination of a pregnancy. 4. Nagele's rule is not always accurate for women with markedly irregular periods that include 1 or more months of amenorrhea.

Absolute contraindications to exercise while pregnant include which of the following? Note: Credit will be given only if all correct and no incorrect choices are selected. Select all that apply. 1. Abruptio placentae 2. Placenta previa after 26 weeks' gestation 3. Preeclampsia-eclampsia 4. Cervical insufficiency (cerclage) 5. Intrauterine growth restriction (IUGR)

Answer: 2, 3, 4 Explanation: 2. Placenta previa after 26 weeks' gestation is an absolute contraindication to exercise. 3. Preeclampsia-eclampsia is an absolute contraindication to exercise. 4. Cervical insufficiency (cerclage) is an absolute contraindication to exercise.

The nurse is preparing a brochure for couples considering pregnancy after the age of 35. Which statements should be included? Note: Credit will be given only if all correct choices and no incorrect choices are selected. Select all that apply. 1. There is a decreased risk of Down syndrome. 2. Preexisting medical conditions can complicate pregnancy. 3. Preterm births are more common. 4. Amniocentesis can be performed to detect genetic anomalies. 5. The increased fertility of women over age 35 makes conception easier.

Answer: 2, 3, 4 Explanation: 2. Preexisting medical conditions, such as hypertension or diabetes, probably play a more significant role than age in maternal well-being and the outcome of pregnancy. 3. The incidence of low-birth-weight infants, preterm births, miscarriage, stillbirth, and perinatal morbidity and mortality is higher among women age 35 or older. 4. Amniocentesis is offered to all women over age 35 to permit the early detection of several chromosomal abnormalities, including Down syndrome; noninvasive analysis of fetal nucleic acid is now commonly recommended to women of advanced maternal age.

A pregnant client complains to the clinic nurse that her varicose veins are causing more discomfort than before. Which recommendations does the nurse make to the client? Note: Credit will be given only if all correct and no incorrect choices are selected. Select all that apply. 1. Increase the time she stands. 2. Walk on a daily basis. 3. Not cross her legs at the knees. 4. Wear support hose. 5. Hyperextend her knee with her feet up.

Answer: 2, 3, 4 Explanation: 2. Regular exercise, such as swimming, cycling, or walking, promotes venous return, which helps prevent varicosities. 3. The client should avoid crossing her legs at the knees because of the pressure it puts on her veins. 4. Supportive hose or elastic stockings may be extremely helpful.

What signs would indicate that a pregnant client's urinalysis culture was abnormal? Note: Credit will be given only if all correct and no incorrect choices are selected. Select all that apply. 1. pH 4.6-8 2. Alkaline urine 3. Cloudy appearance 4. Negative for protein and red blood cells 5. Hemoglobinuria

Answer: 2, 3, 5 Explanation: 2. Alkaline urine could indicate metabolic alkalemia, Proteus infection, or an old specimen. 3. A cloudy appearance could indicate an infection. 5. Hemoglobinuria would be indicated by an abnormal urine color.

During the initial prenatal visit, the nurse assesses the history of the father of the child for which of the following? Note: Credit will be given only if all correct and no incorrect choices are selected. Select all that apply. 1. Stability of living conditions 2. Blood type and Rh type 3. Significant health problems 4. Nutritional history 5. Current use of tobacco

Answer: 2, 3, 5 Explanation: 2. The father of the fetus should be assessed for blood type and Rh factor. 3. The father of the fetus should be assessed for significant health problems. 5. The father of the fetus should be assessed for current alcohol intake, drug use, and tobacco use.

The nurse in the prenatal clinic will tell the client at 38-weeks' gestation to lie on her left side when the client complains of which of the following? Note: Credit will be given only if all correct and no incorrect choices are selected. Select all that apply. 1. Nausea 2. Pallor 3. Clamminess 4. Constipation 5. Dizziness

Answer: 2, 3, 5 Explanation: 2. Vena caval syndrome can cause pallor, which is relieved when the client turns to lie on her left side. 3. Vena caval syndrome can cause clamminess, which is relieved when the client turns to lie on her left side. 5. Vena caval syndrome can cause dizziness, which is relieved when the client turns to lie on her left side.

A nurse is discussing the serving sizes in the grains food group with a new prenatal patient in the clinic. Which food equals one serving size from the grains food group? Note: Credit will be given only if all correct choices and no incorrect choices are selected. Select all that apply. 1. One hamburger roll 2. One slice of bread 3. One cup of pasta 4. One tortilla 5. One ounce of dry cereal

Answer: 2, 4, 5 Explanation: 2. One slice of bread is one serving. 4. One tortilla is one serving. 5. One ounce of dry cereal is one serving.

A client tells the nurse that she does not like citrus fruits, and would like suggestions for alternate vitamin C sources. What should the nurse suggest as good sources of vitamin C? Note: Credit will be given only if all correct choices and no incorrect choices are selected. Select all that apply. 1. Barley and brown rice 2. Strawberries and potatoes 3. Buckwheat and lentils 4. Wheat flour and figs 5. Blueberries and broccoli

Answer: 2, 5 Explanation: 2. Strawberries and potatoes are very good sources of vitamin C.

A pregnant client who is a lacto-vegetarian asks the nurse for assistance with her diet. What instruction should the nurse give? Note: Credit will be given only if all correct choices and no incorrect choices are selected. Select all that apply. 1. "Protein is important; therefore, the addition of one serving of meat a day is necessary." 2. "A daily supplement of vitamin B12 is important." 3. "The high fiber in a vegetarian diet is dangerous for pregnant women." 4. "Eggs are important to add to your diet. Eat six eggs per week." 5. "Milk products contain protein, but they are very low in iron."

Answer: 2, 5 Explanation: 2. Supplementation may be recommended for vegans who have difficulty meeting the recommended amounts of vitamin B12 through food sources.

The nurse assessing a pregnant African American woman in the first trimester understands that a cultural practice is which of the following? 1. Use of herbs like dandelion during pregnancy to increase lactation 2. Drinking ginseng tea for faintness 3. Eating clay to supply dietary minerals 4. Consulting a spiritual advisor to ensure a healthy pregnancy and birth

Answer: 3 Explanation: 3. African American pregnant women may be guided by their extended family into common practices such as geophagia, the ingestion of dirt or clay, which is believed to alleviate mineral deficiencies.

The nurse is assessing a new client in the clinic. The nurse knows that the subjective (presumptive) signs and symptoms of pregnancy include which of the following? 1. Positive urine pregnancy test, enlarged abdomen, and Braxton Hicks contractions 2. Positive urine pregnancy test, amenorrhea, changes in pigmentation of the skin, and softening of the cervix 3. Increase in urination, amenorrhea, fatigue, breast tenderness, and quickening 4. Enlarged abdomen and fetal heartbeat

Answer: 3 Explanation: 3. An increase in urination, amenorrhea, fatigue, breast tenderness, and quickening are all subjective (presumptive) changes of pregnancy.

The nurse is presenting a class of important "dos and don'ts" during pregnancy, including travel considerations. What method of travel does the nurse recommend as most appropriate for a client in her 25th week of pregnancy? 1. Automobile 2. Airplane 3. Train 4. None; this client should not travel

Answer: 3 Explanation: 3. As pregnancy progresses, travel by train is generally recommended for long distances.

Intercourse is contraindicated if the pregnancy is vulnerable because of which diagnosis? 1. Gestational diabetes 2. Cervical insufficiency (cerclage) 3. Abruptio placentae 4. Placenta previa

Answer: 3 Explanation: 3. As the uterus enlarges, the couple will have to experiment with different positions.

What would the nurse include as part of a routine physical assessment for a second-trimester primiparous patient whose prenatal care began in the first trimester and is ongoing? 1. Pap smear 2. Hepatitis B screening (HBsAg) 3. Fundal height measurement 4. Complete blood count

Answer: 3 Explanation: 3. At each prenatal visit, the blood pressure, pulse, and weight are assessed, and the size of the fundus is measured. Fundal height should be increasing with each prenatal visit.

The nurse is listening to the fetal heart tones of a client at 37 weeks' gestation while the client is in a supine position. The client states, "I'm getting lightheaded and dizzy." What is the nurse's best action? 1. Assist the client to sit up. 2. Remind the client that she needs to lie still to hear the baby. 3. Help the client turn onto her left side. 4. Check the client's blood pressure.

Answer: 3 Explanation: 3. During pregnancy the enlarging uterus may put pressure on the vena cava when the woman is supine, resulting in supine hypotensive syndrome. This pressure interferes with returning blood flow and produces a marked decrease in blood pressure with accompanying dizziness, pallor, and clamminess, which can be corrected by having the woman lie on her left side.

During her first months of pregnancy, a client tells the nurse, "It seems like I have to go to the bathroom every 5 minutes." The nurse explains to the client that this is because of which of the following? 1. The client probably has a urinary tract infection. 2. Bladder capacity increases throughout pregnancy. 3. The growing uterus puts pressure on the bladder. 4. Some women are very sensitive to body function changes.

Answer: 3 Explanation: 3. During the first trimester, the growing uterus puts pressure on the bladder, producing urinary frequency until the second trimester, when the uterus becomes an abdominal organ. Near term, when the presenting part engages in the pelvis, pressure is again exerted on the bladder.

The client has delivered her first child at 37 weeks. The nurse would describe this to the client as what type of delivery? 1. Preterm 2. Postterm 3. Early term 4. Near term

Answer: 3 Explanation: 3. Early term births extend from 37 to 38 weeks' gestation.

Which statement, if made by a pregnant client, would indicate that she understands health promotion during pregnancy? 1. "I lie down after eating to relieve heartburn." 2. "I try to limit my fluid intake to 3 or 4 glasses each day." 3. "I elevate my legs while sitting at my desk." 4. "I am avoiding exercise to stay well rested."

Answer: 3 Explanation: 3. Elevating the legs can help decrease lower leg edema.

If a woman has the pre-existing condition of diabetes, the nurse knows that she would be prone to what high-risk factor when pregnant? 1. Vasospasm 2. Postpartum hemorrhage 3. Episodes of hypoglycemia and hyperglycemia 4. Cerebrovascular accident (CVA)

Answer: 3 Explanation: 3. Episodes of hypoglycemia and hyperglycemia would be a high-risk factor for a client with pre-existing diabetes.

What is the increased vascularization causing the softening of the cervix known as? 1. Hegar sign 2. Chadwick sign 3. Goodell sign 4. McDonald sign

Answer: 3 Explanation: 3. Increased vascularization causes the softening of the cervix known as Goodell sign.

The nurse is teaching an early pregnancy class for clients in the first trimester of pregnancy. Which statement by a client requires immediate intervention by the nurse? 1. "When my nausea is bad, I will drink some ginger tea." 2. "The fatigue I am experiencing will improve in the second trimester." 3. "It is normal for my vaginal discharge to be green." 4. "I will urinate less often during the middle of my pregnancy."

Answer: 3 Explanation: 3. Increased whitish vaginal discharge, called leukorrhea, is common in pregnancy. Green discharge is not a normal finding, and indicates a vaginal infection.

A pregnant client confides to the nurse that she is eating laundry starch daily. The nurse should assess the client for which of the following? 1. Alopecia 2. Weight loss 3. Iron deficiency anemia 4. Fecal impaction

Answer: 3 Explanation: 3. Iron deficiency anemia is the most common concern with pica. The ingestion of laundry starch or certain types of clay may contribute to iron deficiency by replacing iron-containing foods from the diet or by interfering with iron absorption.

Which of the following drugs and drug categories can cause multiple fetal central nervous system (CNS), facial, and cardiovascular anomalies? 1. Category C: Zidovudine 2. Category B: Penicillin 3. Category X: Isotretinoin 4. Category A: Vitamin C

Answer: 3 Explanation: 3. Isotretinoin (Accutane), the acne medication, can cause multiple central nervous system (CNS), facial, and cardiovascular anomalies.

The nurse is examining a pregnant woman in the third trimester. What skin changes should the nurse highlight as an alteration for the woman's healthcare provider? 1. Linea nigra 2. Melasma gravidarum 3. Petechiae 4. Vascular spider nevi

Answer: 3 Explanation: 3. Petechiae are pinpoint red or purple spots on the skin. They are seen in hemorrhagic conditions.

The prenatal client in her third trimester tells the clinic nurse that she works 8 hours a day as a cashier and stands when at work. What response by the nurse is best? 1. "No problem. Your baby will be fine." 2. "Do you get regular breaks for eating?" 3. "Your risk of preterm labor is higher." 4. "Standing might increase ankle swelling."

Answer: 3 Explanation: 3. Pregnant women who are employed in jobs that require prolonged standing (more than 3 hours) do have a higher incidence of preterm birth.

A client with a normal prepregnancy weight asks why she has been told to gain 25-35 pounds during her pregnancy while her underweight friend was told to gain more weight. What should the nurse tell the client the recommended weight gain is during pregnancy? 1. 25-35 pounds, regardless of a client's prepregnant weight 2. More than 25-35 pounds for an overweight woman 3. Up to 40 pounds for an underweight woman 4. The same for a normal weight woman as for an overweight woman

Answer: 3 Explanation: 3. Prepregnant weight determines the recommended weight gain during pregnancy. Underweight women are advised to gain 28-40 pounds.

The nurse has received a phone call from a multigravida who is 21 weeks pregnant and has not felt fetal movement yet. What is the best action for the nurse to take? 1. Reassure the client that this is a normal finding in multigravidas. 2. Suggest that she should feel for movement with her fingertips. 3. Schedule an appointment for her with her physician for that same day. 4. Tell her gently that her fetus is probably dead.

Answer: 3 Explanation: 3. Quickening, or the mother's perception of fetal movement, occurs about 18 to 20 weeks after the LMP in a primigravida (a woman who is pregnant for the first time) but may occur as early as 16 weeks in a multigravida (a woman who has been pregnant more than once).

Screening for gestational diabetes mellitus (GDM) is typically completed between which of the following weeks of gestation? 1. 36 and 40 weeks 2. Before 20 weeks 3. 24 and 28 weeks 4. 30 and 34 weeks

Answer: 3 Explanation: 3. Screening for gestational diabetes mellitus (GDM) is typically completed between 24 and 28 weeks' gestation.

During a non-stress test, the nurse notes that the fetal heart rate decelerates about 15 beats during a period of fetal movement. The decelerations occur twice during the test, and last 20 seconds each. The nurse realizes these results will be interpreted as which of the following? 1. A negative test 2. A reactive test 3. A nonreactive test 4. An equivocal test

Answer: 3 Explanation: 3. The FHR acceleration must be at least 15 beats per minute above baseline for at least 15 seconds from baseline to baseline. A nonreactive NST is one that lacks sufficient FHR accelerations over a 40-minute period.

The pregnant client cannot tolerate milk or meat. What would the nurse recommend to the client to assist in meeting protein needs? 1. Wheat bread and pasta 2. Ice cream and peanut butter 3. Eggs and tofu 4. Beans and potatoes

Answer: 3 Explanation: 3. The best food choices that are nondairy and complete proteins alone are eggs and tofu.

The nurse knows that a lecithin/sphingomyelin (L/S) ratio finding of 2:1 on amniotic fluid means which of the following? 1. Fetal lungs are still immature. 2. The fetus has a congenital anomaly. 3. Fetal lungs are mature. 4. The fetus is small for gestational age.

Answer: 3 Explanation: 3. The concentration of lecithin begins to exceed that of sphingomyelin, and at 35 weeks the L/S ratio is 2:1. When at least two times as much lecithin as sphingomyelin is found in the amniotic fluid, RDS is very unlikely.

Which of the following tests has become a widely accepted method of evaluating fetal status? 1. Contraction stress test (CST) 2. MSAFP test 3. Non-stress test (NST) 4. Nuchal translucency test

Answer: 3 Explanation: 3. The non-stress test (NST) has become a widely accepted method of evaluating fetal status. This test involves using an external electronic fetal monitor to obtain a tracing of the fetal heart rate (FHR) and observation of acceleration of the FHR with fetal movement.

The nurse is providing guidance for a woman in her second trimester of pregnancy and telling her about some of the signs and symptoms that she might experience. Which statement by the client indicates that further teaching is necessary? 1. "During the third trimester, I might have frequent urination." 2. "During the third trimester, I might have heartburn." 3. "During the third trimester, I might have back pain." 4. "During the third trimester, I might have a persistent headache."

Answer: 3 Explanation: 3. The nurse responds to this most critical statement because of the danger of fetal alcohol syndrome.

The pregnant client in her second trimester states, "I didn't know my breasts would become so large. How do I find a good bra?" The best answer for the nurse to make would be which of the following? 1. "Avoid cotton fabrics and get an underwire bra; they fit everyone best." 2. "Just buy a bra one cup size bigger than usual, and it will fit." 3. "Look for wide straps and cups big enough for all of your breast tissue." 4. "There isn't much you can do for comfort. Try not wearing a bra at all."

Answer: 3 Explanation: 3. The nurse should instruct the client to get a bra that fits with straps that are wide and do not stretch and a cup that holds all breast tissue comfortably.

The nurse in a prenatal clinic finds that four clients have called with complaints related to their pregnancies. Which call should the nurse return first? 1. Pregnant woman at 7 weeks' gestation reporting nasal stuffiness 2. Pregnant woman at 38 weeks' gestation experiencing rectal itching and hemorrhoids 3. Pregnant woman at 15 weeks' gestation with nausea and vomiting and a 15-pound weight loss 4. Pregnant woman at 32 weeks' gestation treating constipation with prune juice

Answer: 3 Explanation: 3. The nurse should return this call first because this patient is the highest priority. A 15-pound weight loss is not an expected finding. Although some nausea is common, the woman who suffers from extreme nausea coupled with vomiting requires further assessment.

A client comes into the prenatal clinic accompanied by her boyfriend. When asked by the nurse why she is there, the client looks down, and the boyfriend states, "She says she is pregnant. She constantly complains of feeling tired, and her vomiting is disgusting." What is a priority for the nurse to do at this point? 1. Ask the client what time of the day her fatigue is more common. 2. Recommend that the woman have a pregnancy test done as soon as possible. 3. Continue the interview of the client in private. 4. Give the woman suggestions on ways to decrease the vomiting.

Answer: 3 Explanation: 3. The nurse should suspect that the client is in an abusive relationship. The priority is for the nurse to get the client away from the boyfriend and continue the interview.

A pregnant client has a hemoglobin of 10 g/dL and a Hct of 30%. The clinic nurse recognizes the fetus is at risk for which of the following? Note: Credit will be given only if all correct and no incorrect choices are selected. Select all that apply. 1. Macrosomia 2. Respiratory distress syndrome 3. Low birth weight 4. Prematurity 5. Fetal death

Answer: 3, 4, 5 Explanation: 3. Anemia places the fetus at risk for a low birth weight. 4. Anemia places the fetus at risk for premature birth. 5. Anemia places the fetus at risk for fetal death.

The nurse is presenting a prenatal class to a group of women pregnant for the first time who are all over 35 years of age. The nurse knows that the advantage of waiting until later to start a family is which of the following? 1. That the woman will have an easier labor and delivery. 2. That the baby will be at less of a risk for congenital anomalies. 3. That the woman is more likely to be financially secure. 4. That the woman will be more fertile than a younger woman would.

Answer: 3 Explanation: 3. This delay in starting a family allows women to pursue advanced educational degrees and prepare financially for the impact children will have on their lives.

The client in the prenatal clinic tells the nurse that she is sure she is pregnant because she has not had a menstrual cycle for 3 months, and her breasts are getting bigger. What response by the nurse is best? 1. "Lack of menses and breast enlargement are presumptive signs of pregnancy." 2. "The changes you are describing are definitely indicators that you are pregnant." 3. "Lack of menses can be caused by many things. We need to do a pregnancy test." 4. "You're probably not pregnant, but we can check it out if you like."

Answer: 3 Explanation: 3. This is a true statement, and addresses that these changes could be caused by conditions other than pregnancy.

The nurse is preparing a client in her second trimester for a three-dimensional ultrasound examination. Which statement indicates that teaching has been effective? 1. "If the ultrasound is normal, it means my baby has no abnormalities." 2. "The nuchal translucency measurement will diagnose Down syndrome." 3. "I might be able to see who the baby looks like with the ultrasound." 4. "Measuring the length of my cervix will determine whether I will deliver early."

Answer: 3 Explanation: 3. Three-dimensional ultrasound uses algorithms to vary opacity, transparency, and depth to project an image. This allows curved structures such as the fetal face to be viewed.

The client with an abnormal quadruple screen is scheduled for an ultrasound. Which statement indicates that the client understands the need for this additional antepartal fetal surveillance? 1. "After the ultrasound, my partner and I will decide how to decorate the nursery." 2. "During the ultrasound we will see which of us the baby looks like most." 3. "The ultrasound will show whether there are abnormalities with the baby's spine." 4. "The blood test wasn't run correctly, and now we need to have the sonogram."

Answer: 3 Explanation: 3. Ultrasound is used to detect neural tube defects. An abnormal serum quadruple screen is not the result of a lab error, and can indicate either an open neural tube defect or trisomy 18 or 21.

The nurse is caring for a pregnant client. The client's husband has come to the prenatal visit. Which question is the best for the nurse to use to assess the father's adaptation to the pregnancy? 1. "What kind of work do you do?" 2. "What furniture have you gotten for the baby?" 3. "How moody has your wife been lately?" 4. "How are you feeling about becoming a father?"

Answer: 4 Explanation: 4. A husband's adaptation to pregnancy includes his feelings about impending fatherhood.

The nurse is returning phone calls from clients. Which client does the nurse anticipate would not require a serum beta hCG? 1. A client with a risk of ectopic pregnancy 2. A client with spotting during pregnancy 3. A client with previous pelvic inflammatory disease 4. A client with a previous history of twins

Answer: 4 Explanation: 4. A previous history of twins is not a risk factor for ectopic pregnancy. Beta hCG testing is not indicated for this client.

Each of the following pregnant women is scheduled for a 14-week antepartal visit. In planning care, the nurse would give priority teaching on amniotic fluid alpha-fetoprotein (AFP) screening to which client? 1. 28-year-old with history of rheumatic heart disease 2. 18-year-old with exposure to HIV 3. 20-year-old with a history of preterm labor 4. 35-year-old with a child with spina bifida

Answer: 4 Explanation: 4. Alpha-fetoprotein (AFP) is a fetal protein that is excreted from the fetal yolk sac during the first 6 weeks of pregnancy. AFP levels can be high or low, with each having different implications for the fetus. If the fetus has a neural tube defect (NTD), the AFP levels will be elevated. NTDs can range from anencephaly to spina bifida. With a past history of a child with spina bifida, this client would be strongly encouraged to have the AFP screening.

The nurse is assessing a newly pregnant client. Which finding does the nurse note as a normal psychosocial adjustment in this client's first trimester? 1. An unlisted telephone number 2. Reluctance to tell the partner of the pregnancy 3. Parental disapproval of the woman's partner 4. Ambivalence about the pregnancy

Answer: 4 Explanation: 4. Ambivalence toward a pregnancy is a common psychosocial adjustment in early pregnancy.

When preparing nutritional instruction, which pregnant client would the nurse consider the highest priority? 1. 40-year-old gravida 2 2. 22-year-old primigravida 3. 35-year-old gravida 4 4. 15-year-old nulligravida

Answer: 4 Explanation: 4. An expectant adolescent must meet the nutritional needs for her own growth in addition to the nutritional needs of pregnancy.

Which third-trimester client would the nurse suspect might be having difficulty with psychological adjustments to her pregnancy? 1. A woman who says, "Either a boy or a girl will be fine with me" 2. A woman who puts her feet up and listens to some music for 15 minutes when she is feeling too stressed 3. A woman who was a smoker but who has quit at least for the duration of her pregnancy 4. A woman who has not investigated the kind of clothing or feeding methods the baby will need

Answer: 4 Explanation: 4. By the third trimester, the client should be planning and preparing for the baby (for example, living arrangements, clothing, feeding methods).

A pregnant woman is having a nipple-stimulated contraction stress test. Which result indicates hyperstimulation? 1. The fetal heart rate decelerates when three contractions occur within a 10-minute period. 2. The fetal heart rate accelerates when contractions last up to 60 seconds. 3. There are more than five fetal movements in a 10-minute period. 4. There are more than three uterine contractions in a 6-minute period.

Answer: 4 Explanation: 4. Hyperstimulation is characterized by contractions that occur more frequently than every 2 minutes or last longer than 90 seconds.

The partner of a client at 16 weeks' gestation accompanies her to the clinic. The partner tells the nurse that the baby just doesn't seem real to him, and he is having a hard time relating to his partner's fatigue and food aversions. Which statement would be best for the nurse to make? 1. "If you would concentrate harder, you'd be aware of the reality of this pregnancy." 2. "My husband had no problem with this. What was your childhood like?" 3. "You might need professional psychological counseling. Ask your physician." 4. "Many men feel this way. Feeling the baby move in a few weeks will help make it real to you."

Answer: 4 Explanation: 4. Initially, expectant fathers may have ambivalent feelings.The extent of ambivalence depends on many factors, including the father's relationship with his partner, his previous experience with pregnancy, his age, his economic stability, and whether the pregnancy was planned. The expectant father must first deal with the reality of the pregnancy and then struggle to gain recognition as a parent from his partner, family, friends, coworkers, society-and from his baby as well.

The nurse working with mothers over 35 having their first baby knows there are some disadvantages. For what disadvantage would the nurse carefully assess in each client? 1. What kind of insurance the client has for maternity care 2. Whether the client is married 3. Whether the client will continue working after the baby arrives 4. Whether the client has any chronic disease that will have to be addressed

Answer: 4 Explanation: 4. It is important for the nurse to question and assess for any chronic illnesses. The risk of pregnancy complications is higher in women over age 35 who have a chronic condition such as hypertension or diabetes, or who are in poor general health.

A woman calls the clinic and tells a nurse that she thinks she might be pregnant. She wants to use a home pregnancy test before going to the clinic, and asks the nurse how to use it correctly. What information should the nurse give? 1. The false-positive rate of these tests is quite high. 2. If the results are negative, the woman should repeat the test in 2 weeks if she has not started her menstrual period. 3. A negative result merely indicates growing trophoblastic tissue and not necessarily a uterine pregnancy. 4. The client should follow up with a healthcare provider after taking the home pregnancy test.

Answer: 4 Explanation: 4. It is important that clients remember that the tests are not always accurate and they should follow up with a healthcare provider.

A woman at 28 weeks' gestation is asked to keep a fetal activity record and to bring the results with her to her next clinic visit. One week later, she calls the clinic and anxiously tells the nurse that she has not felt the baby move for more than 30 minutes. Which of the following would be the nurse's most appropriate initial comment? 1. "You need to come to the clinic right away for further evaluation." 2. "Have you been smoking?" 3. "When did you eat last?" 4. "Your baby might be asleep."

Answer: 4 Explanation: 4. Lack of fetal activity for 30 minutes typically is insignificant. Movement varies considerably, but most women feel fetal movement at least 10 times in 3 hours.

The nurse is working with a pregnant 14-year-old. Which statement indicates that additional education is required? 1. "Because I am still growing, I need more calories than a pregnant adult." 2. "I need to eat fruit and vegetables every day to get enough vitamins." 3. "My favorite food is pizza, and I eat it once a week." 4. "Because I don't eat breakfast, I'll have to eat more at supper."

Answer: 4 Explanation: 4. Pregnant young adolescents should eat breakfast to ensure the adequate calorie and protein intake needed. In assessing the diet of the pregnant adolescent, the nurse should consider the eating pattern over time, not simply a single day's intake.

The clinic nurse is compiling data for a yearly report. Which client would be classified as a primigravida? 1. A client at 18 weeks' gestation who had a spontaneous loss at 12 weeks 2. A client at 13 weeks' gestation who had an ectopic pregnancy at 8 weeks 3. A client at 14 weeks' gestation who has a 3-year-old daughter at home 4. A client at 15 weeks' gestation who has never been pregnant before

Answer: 4 Explanation: 4. Primigravida means a woman who is pregnant for the first time.

The nurse is providing health teaching to a group of women of childbearing age. One woman states that she is a smoker, and asks about the effect of smoking on her fetus. The nurse tells her that which fetal complication can occur when the mother smokes? 1. Genetic changes in the fetal reproductive system 2. Extensive central nervous system damage 3. Addiction to the nicotine inhaled from the cigarette 4. Low birth rate

Answer: 4 Explanation: 4. Smoking can cause low birth rate.

Which of the following is common in many non-Western cultures and is on the increase in the United States? 1. Ceremonial rituals and rites 2. Cultural assessment 3. Cultural values 4. Cosleeping

Answer: 4 Explanation: 4. Some parents advocate cosleeping or bed sharing (one or both parents sleeping with their baby or young child). Cosleeping, which is common in many non-Western cultures, is on the increase in the United States.

The nurse is assessing a client who is at 35 weeks' gestation. What does the nurse expect the client to report at this phase of pregnancy? 1. Nausea and vomiting 2. Maternal ambivalence 3. Emotional shifts from highs to lows 4. Stretch marks on the abdomen

Answer: 4 Explanation: 4. Striae are purplish stretch marks that may develop as the pregnancy progresses.

The primigravida at 22 weeks' gestation has a fundal height palpated slightly below the umbilicus. Which of the following statements would best describe to the client why she needs to be seen by a physician today? 1. "Your baby is growing too much and getting too big." 2. "Your uterus might have an abnormal shape." 3. "The position of your baby can't be felt." 4. "Your baby might not be growing enough."

Answer: 4 Explanation: 4. The fundal height at 20-22 weeks should be about even with the umbilicus. At 22 weeks' gestation, a fundal height below the umbilicus and the size of the uterus that is inconsistent with length of gestation could indicate fetal demise.

A client in her third trimester of pregnancy reports frequent leg cramps. What strategy would be most appropriate for the nurse to suggest? 1. Point the toes of the affected leg 2. Increase intake of protein-rich foods 3. Limit activity for several days 4. Flex the foot to stretch the calf

Answer: 4 Explanation: 4. The nurse should advise the client to practice dorsiflexion of feet to stretch affected muscle.

The nurse is working in an outpatient clinic. Which client's indications most warrant fetal monitoring in the third trimester? 1. Gravida 4, para 3, 39 weeks, with a history of one spontaneous abortion at 8 weeks 2. Gravida 1, para 0, 40 weeks, with a history of endometriosis and a prior appendectomy 3. Gravida 3, para 2, with a history of gestational diabetes controlled by diet 4. Gravida 2, para 1, 36 weeks, with a history of history of preterm labor or cervical insufficiency

Answer: 4 Explanation: 4. The preterm client with a history of preterm labor or cervical insufficiency needs close monitoring for preterm labor onset.

The client at 30 weeks' gestation with her first child is upset. She tells the prenatal clinic nurse that she is excited to become a mother, and has been thinking about what kind of parent she will be. But her mother has told her that she doesn't want to be a grandmother because she doesn't feel old enough, while her husband has said that the pregnancy doesn't feel real to him yet, and he will become excited when the baby is actually here. What is the most likely explanation for what is happening within this family? 1. Her husband will not attach with this child and will not be a good father. 2. Her mother is rejecting the role of grandparent, and will not help out. 3. The client is not progressing through the developmental tasks of pregnancy. 4. The family members are adjusting to the role change at their own paces.

Answer: 4 Explanation: 4. This is a true statement. With each pregnancy, routines and family dynamics are altered, requiring readjustment and realignment.


Kaugnay na mga set ng pag-aaral

Integumentary Assessment/Problems

View Set

HRT 390- Chapter 9, 10, 11, 12, 14

View Set

Chemistry IGCSE - topic 1 - principles of chemistry

View Set

FL 2-15 taxes, retirement, and other insurance concepts

View Set

Viaje a la semilla por Alejo Carpentier

View Set

environmental science chapter 13

View Set